[seqfan] Re: A MathOverflow question related to A000081

David Corneth davidacorneth at gmail.com
Wed Feb 27 09:20:18 CET 2019


Could it be that the somewhat steaper part is arround 1/e =
0.3678794411714.... because of some property of 1/e?


On Wed, Feb 27, 2019 at 1:50 AM Alex Meiburg <timeroot.alex at gmail.com>
wrote:

> My first thought would be to see *which* sequences appear where in the
> sorted version. Perhaps then you can deduce something about which functions
> are responsible for what parts of the graph.
>
> On Tue, Feb 26, 2019, 4:41 PM Vladimir Reshetnikov <
> v.reshetnikov at gmail.com>
> wrote:
>
> > I asked a question at MathOverflow related to https://oeis.org/A000081:
> > https://mathoverflow.net/questions/324203/integrals-of-power-towers
> >
> > Please share your thoughts there or in this mailing list.
> >
> > --
> > Best regards
> > Vladimir Reshetnikov
> >
> > --
> > Seqfan Mailing list - http://list.seqfan.eu/
> >
>
> --
> Seqfan Mailing list - http://list.seqfan.eu/
>



More information about the SeqFan mailing list